Editor:  Many candidates say that if elected they will reduce governmental intrusion into voters' lives. But voters a...

mgraves5 on August 23, 2019

How does A strengthen

A makes perfect sense if the question was a weaken question or would strengthen the argument if the conclusion was that intrusion would be reduce over time. As I interpret the conclusion it says that intrusion will rarely be reduced over time. How will politicians keeping their promises make that stronger? I would think that if they kept their promises intrusion would be reduced over time, not rarely reduced. My only thought is that if they keep their promise to provide assistance and assistance is considered intrusion then that would make sense. Seems like a very tricky question, but clearly 3 answers could be eliminated.

Replies
Create a free account to read and take part in forum discussions.

Already have an account? log in

Ravi on August 23, 2019

@mgraves5,

Let's look at (A).

In the stimulus, the editor tells us that some politicians promise
that they will get the government to reduce their intrusion into
people's lives. However, voters actually vote for politicians who
promise that government will help solve their most pressing problems.
This assistance costs money, and money can only come from taxes. To
top it off, taxes are a type of government intrusion. The editor then
concludes that governmental intrusion into the lives of voters will
rarely be substantially reduced over time in a democracy.

There are two large assumptions that the argument is making. The first
is that the argument is assuming that the politicians will fulfill
their promises, as the premise states that voters elect politicians
who promise assistance. Additionally, taxes are simply one type of
governmental intrusion. The argument is assuming that other types of
governmental intrusion will not be reduced to compensate for any
increase in taxes. The correct answer choice will likely fill in one
of these gaps.

(A) says, "Politicians who win their elections usually keep their
campaign promises."

(A) looks great. We know that voters elect politicians who promise to
give assistance. (A) tells us that the politicians will fulfill their
promises, which means that higher taxes and additional intrusion are
far more likely. Thus, (A) is the correct answer choice, as it fills
in the gap of one of the two big assumptions in the argument.

Does this make sense? Let us know if you have any other questions!

mgraves5 on August 24, 2019

It makes more sense now. Thanks!